LSAT and Law School Admissions Forum

Get expert LSAT preparation and law school admissions advice from PowerScore Test Preparation.

User avatar
 Dave Killoran
PowerScore Staff
  • PowerScore Staff
  • Posts: 5852
  • Joined: Mar 25, 2011
|
#78451
Complete Question Explanation
(The complete setup for this game can be found here: lsat/viewtopic.php?t=15592)

The correct answer choice is (A)

The question asks you to identify the minimum number of machines purchased in 1987. To answer this question, you must optimize the setup in a way that avoids 1987 as much as possible. As only the machines in offices 3 and 4 have any possibility of a machine purchased in 1987, start with those two offices and attempt to avoid purchases in 1987. As shown in the hypothetical below, the computer in office 4, the printer in office 4, and the computer in office 3 can all be purchased in 1988, resulting in no machines purchased in 1987:
PT1-Jun1991 game 2 #10 diagram 1.png
Accordingly, the correct answer is 0, answer choice (A).
You do not have the required permissions to view the files attached to this post.

Get the most out of your LSAT Prep Plus subscription.

Analyze and track your performance with our Testing and Analytics Package.